0 Daumen
518 Aufrufe

Aufgabe:

\(\text{Die "Floor"Funktion ist definiert durch}\\ \space \space \space \space \space \space \space \space \space \lfloor x\rfloor = max\lbrace n \in \Z\text{ | n} \leq x\rbrace\\ \text{Die Funktion f sei definiert durch}\\ \space \space \space \space \space \space \space \space \space f\text{ : }\R\rightarrow \R\text{, x}\mapsto\begin{cases} \lfloor \frac{1}{x}\rfloor \cdot x^2, & \text{falls }x\neq0\\ 0, & \text{falls }x= 0 \end{cases}\\ \text{Zeigen Sie, dass }f \text{ in allen Punkten der Form }x_0=\frac{1}{n}\text{, n}\in \Z \backslash \lbrace 0\rbrace \text{ unstetig ist.}\)


Problem/Ansatz:

Hallo, für die Lösung der Aufgabe habe ich bisher 2 Verfahren, die wir in der Vorlesung behandelt haben, ausgetestet.

Das wäre einmal das ε-δ-Kriterium, bei dem ich nach einem ε > 0 suche, sodass für alle δ > 0 ein x existiert, sodass gilt

$$\space \space \space \space \space \space \space \space \space|x-x_0| < \delta \space\land |f(x) - f(x_0)| \geq \epsilon$$

Um nachzuvollziehen was bei der Funktion eigentlich passiert, habe ich mir das mal aufgemalt und gesehen, dass der Sprung an den unstetigen Stellen gegen $$\space \space \space \space \space \space \space \space \space(\frac{1}{n+1})^2$$ laufen müsste. Also habe ich gesagt sei

$$\space \space \space \space \space \space \space \space \space\epsilon=\frac{1}{2n^2}$$ und dafür wollte ich mir dann ein passendes x raussuchen. Da komme ich aber auch nach langem Ausprobieren nicht mehr weiter, weil nicht sehe wie ich dieses x von δ abhängig machen kann und gleichzeitig sicherstellen, dass $$\space \space \space \space \space \space \space \space \space|f(x) - f(x_0)| \geq \epsilon$$ erfüllt ist.


Das andere Verfahren mit Limes, bei dem ich Unstetigkeit in x0 zeigen kann indem ich $$\space \space \space \space \space \space \space \space \space\lim\limits_{x\to x_0}f(x)\neq f(x_0)$$ zeige, bringt mich auch nicht weiter. Ich denke die Auswahl meiner Folge ist da einfach nicht gut.

Ich habe im Internet bereits einen Lösungsweg gefunden, den ich nicht nachvollziehen kann und dessen Voraussetzungen wir in der Vorlesung nicht behandelt haben. Ich glaube es ging dabei um linksseitigen und rechtsseitigen Limes, aber ich möchte die Aufgabe eben mit den Möglichkeiten der Vorlesung lösen.
Was ich suche ist z.B. ein Tipp wie man x findet, Hinweise wo ich bisher schon falsch liege oder allgemein Vorschläge diese Aufgabe zu lösen.

Avatar von

1 Antwort

0 Daumen
 
Beste Antwort

Hallo,

eigentlich hast Du alles schon zusammen, was Du brauchst. Ich komme bei der Darstellung von f auf ein etwas anderes Ergebnis, deshalb schreibe ich das mal hier auf - musst Du noch mal checken:

$$f(x)=nx^2 \text{ für } \frac{1}{n+1} < x \leq \frac{1}{n}$$

und entsprechend:

$$f(x)=(n-1)x^2 \text{ für } \frac{1}{n} < x \leq \frac{1}{n-1}$$

Für \(x_0=\frac{1}{n}\) ist damit \(f(x_0)=\frac{1}{n}\). Wir betrachten jetzt eine Folge \(x_k)\) mit

$$\frac{1}{n} < x_k \leq \frac{1}{n-1} \text{ und } x_k \to x_0$$

Für eine solche Folge gilt:

$$f(x_k)=(n-1)x_k^2 \to \frac{n-1}{n^2} \neq f(x_0).$$

Für die Sprunghöhe erhalte ich dann:

$$\frac{1}{n}-\frac{n-1}{n^2}=\frac{1}{n^2}$$

Ist also anders als bei Dir, musst du deshalb nochmal nachrechnen.

Wenn Du das mit dem \(\epsilon- \delta\)-Kriterium aufschreiben willst, musst Du untersuchen, für welche x im Intervall \(]\frac{1}{n},\frac{1}{n-1}]\) die Ungleichung

$$\frac{1}{n}-(n-1)x^2 \geq \epsilon = \frac{1}{2n^2} (\text{ zum Beispiel})$$

Das ist zwar nicht sehr elegant, aber es geht.

Gruß

Avatar von 13 k

Hallo,

vielen, vielen Dank erstmal für die ausführliche Antwort, ich finde es wirklich toll wie vielen Menschen hier beim Lernen geholfen wird.

Beim Durchgehen Ihrer Lösung habe ich schnell meinen Fehler entdeckt, und zwar bin ich bei der Sprunghöhe wohl auf der x-Achse verrutscht und habe mir bei Betrachtung von n den Sprung bei n+1 angeschaut.

Darüber hinaus ist Ihre Darstellung sehr hilfreich, weil ich sehe, dass es schon mal ein richtiger Schritt von mir war die Funktion aufzumalen und sozusagen erst einmal nach einer intuitiven Lösung zu suchen. Denn das was Sie aufgeschrieben haben ist eigentlich genau das, was man intuitiv vielleicht sieht, wenn man sich die Funktion anschaut. Ich merke also ich muss an dieser Stelle noch gründlicher sein und das ganz präzise aufschreiben und wenn ich später nicht weiter komme nochmal meine Grundannahmen auf Korrektheit überprüfen.

Also Dankeschön nochmal, die Aufgabe hat mich wirklich lange beschäftigt!

Gern geschehen, viel Erfolg weiterhin

Gruß

Ein anderes Problem?

Stell deine Frage

Willkommen bei der Mathelounge! Stell deine Frage einfach und kostenlos

x
Made by a lovely community